M5: Joinder & Supplemental Jurisdiction Flashcards

1
Q

Explain FRCP 18(a) and the logic behind it

A

Allows a P to assert any claims she has against an opponent, whether related or unrelated

Why? If the parties are already in court, represented by counsel, ready to settle their differences, why not do them all at the same time? Doesn’t worry about parties litigating over whether or not a case is related.

How well did you know this?
1
Not at all
2
3
4
5
Perfectly
2
Q

P brings a suit against D for damage to her house during a construction job and joins completely unrelated claims against D for damages in an auto accident she had with D last year. How will these claims be tried?

A

FRCP 18 allows P to bring together both claims in a single suit, the facts and legal issues are different. Evidence and witnesses needed to prove the claims is also different. B/c hearing these claims together would be confusing for a jury and waste time, the trial judge could authorize separate trials under FRCP 42(b)

How well did you know this?
1
Not at all
2
3
4
5
Perfectly
3
Q

Under FRCP 20, P gets control over what other P’s and D’s they want to join in a suit. Why not require parties who have claims based on a single transaction or occurrence to sue together?

A

Requiring joinders would be more efficient for a court, but also problematic for P’s. What if all of the defendants involved were not subject to PJ in the same state? Or if all but one were subject to PJ in a state, and the one was only subject to PJ in a completely different state? It would be more inconvenient for the person bringing the suit.

What if some of the D’s were not diverse? Forcing a joinder would prevent P from suing in federal court. This restricts P’s choices.

P’s joinder may restrict their choice of court, lawyer, D’s to sue, and timing.

How well did you know this?
1
Not at all
2
3
4
5
Perfectly
4
Q

What is the two-pronged approach for joinders under FRCP 20?

A

Limiting P’s choices

(a) Two part test: P’s can sue together if they assert claims that
“arises out of the same transaction, occurrence, or series of transactions or occurrences; and
If their claims involve any question of law or fact common to all P’s”

(a)(2) D may be sued together if the above criteria are met.

How well did you know this?
1
Not at all
2
3
4
5
Perfectly
5
Q

P was assaulted by a police officer. Based on the investigation, he concludes that it was either D1 or D2 (both officers) who committed the assault.

Can he sue them both under FRCP 20(a)(1)? If so, can he recover from both D1 and D2?

A

Yes to suing both, because P seeks relief from both D1 and D2 arising from the same assault. His claim has a common question: who assaulted him?

No to recovery from both. He was assaulted by only one of them.

How well did you know this?
1
Not at all
2
3
4
5
Perfectly
6
Q

Dow Chemical Company discharges chemicals into a stream over a period of time. Farmer Pat wants to recover from damages to his farmland cause by the chemicals during flooding. Citizen Parker wants to recover for an illness he claims resulted from drinking water polluted by Dow. Farmer Petunia has not suffered any damage, but wants to avoid further damage to her land by getting an injunction to prevent Dow from continuing to release chemicals into the stream.

May the three of them join as co plaintiffs under FRCP 20?

A

Yes, all three P’s may sue for different remedies but still join under FRCP 20(a)(1). All of these claims arise from the discharge of chemicals by D.

There will likely be some common question of fact or law, like “did the chemicals come from Dow’s plant?” or “how much chemicals did Dow release over time and is that beyond what’s legal?”

This is efficient in the long run.

How well did you know this?
1
Not at all
2
3
4
5
Perfectly
7
Q

Explain why a Plaintiff or Defendant would prefer or try to dismiss a joinder under FRCP 20.

A

Plaintiffs = prefer efficiency, bolster the suit with multiple/repeated facts piled together

Defendants = keep it separate, avoid the impact of repeated adverse testimony

How well did you know this?
1
Not at all
2
3
4
5
Perfectly
8
Q

True or false: It is possible for a FRCP 20 joinder to be allowed by a court early in a case, then sever the joinder later?

A

True! Though it’s not a severance via FRCP, but instead through FRCP 42(b).

How well did you know this?
1
Not at all
2
3
4
5
Perfectly
9
Q

Patrick is arrested by Officer Danbury while in the City of Atlantis. Patrick claims that Danbury used excessive force in arresting him and sues D and the City of Atlantis for violation of his constitutional rights.

Patrick has properly joined Danbury and the City under FRCP 20(a). However, under federal law, City of Atlantis would only be liable for Danbury’s acts if had a policy or custom of tolerating/encouraging excessive force in arrest. Thus, Patrick has the burden of proving that City had a policy or custom to that effect.

Would it be beneficial to keep the joined claims or to sever? To which party?

A

Officer Danbury may not want to have the claims tried together, since the burden for proving wrongdoing for the City includes finding a pattern of cases where excessive force was used. If there’s multiple cases, Danbury could get lumped in with the rest of the offending officers.

The City may want separate trials, and have Danbury go first. If Patrick can’t prove excessive force, have the trial against the City dismissed.

Patrick may want to keep these together since it shows that the City and its officers have a pattern of excessive force. Compounds to his case.

How well did you know this?
1
Not at all
2
3
4
5
Perfectly
10
Q

True or false: Right to join parties under FRCP 20 supersedes any need for PJ or SMJ

A

False. The right to join parties under FRCP does not confer subject matter or personal jurisdiction. These are very different concepts.

How well did you know this?
1
Not at all
2
3
4
5
Perfectly
11
Q

P (from Oregon) sues D1 (from California) and D2 (from Oregon) for injuries suffered in an auto accident in Oregon. P brings the suit in Federal District Court of the Eastern District of Oregon (where the accident happened), seeking 200K in damages.

Is the joinder proper under FRCP 20(a)(2)? May the court hear the case?

A

Joinder is proper because P is suing D1 and D2 for the same occurence and there will be common questions of fact regarding the accident.

SMJ is lacking, however, because there is no federal claim and diversity is lacking.

Thus, the court cannot hear the case.

How well did you know this?
1
Not at all
2
3
4
5
Perfectly
12
Q

P (from Oregon) sues D1 (From California) and D2 (from Idaho), seeking $200K in damages for injuries P suffered in an accident in Oregon. She brings action in a federal court in California.

Is joinder proper under FRCP 20(a)(2)? May the court hear the case?

A

Joinder is proper, but the court lacks personal jurisdiction over D2 (from Idaho) for the claim, which arises out of an Oregon accident. D2 would be dismissed from the claim.

How well did you know this?
1
Not at all
2
3
4
5
Perfectly
13
Q

True or false: Compulsory counterclaims can be brought at any time.

A

False. FRCP 13
Compulsory = must be asserted in the same action, failure to do so would be considered a waiver if there was time and understanding from P to plead the counterclaim.

How well did you know this?
1
Not at all
2
3
4
5
Perfectly
14
Q

What is the transaction-or-occurrences test in counterclaim?

A

Failure to assert a compulsory counterclaim constitutes waiver, and the party will be estopped from later asserting the claim if she was aware of the claim and had an opportunity to file before settlement.

Did the claim arise out of the same transaction?
+ Common issues in question?
+ Same evidence underpinning both claims?
+ Logical relationship between claims?
+ Logical relationship between types of claims and desired remedies?

How well did you know this?
1
Not at all
2
3
4
5
Perfectly
15
Q

True or false: FRCP 13 allows additional parties to be joined in a crossclaim

A

True.

13(h) expressly authorizes adding additional parties to both crossclaims and counterclaims. Defending parties can bring a stranger into the lawsuit — someone who the P did not choose.

How well did you know this?
1
Not at all
2
3
4
5
Perfectly
16
Q

True or false: Plaintiffs cannot assert crossclaims under FRCP 13(g)

A

False. P’s can also assert crossclaims under 13(g).

How well did you know this?
1
Not at all
2
3
4
5
Perfectly
17
Q

True or false: many impleader claims under FRCP 14(a) are brought for contribution

A

True. Erkins is an example.

How well did you know this?
1
Not at all
2
3
4
5
Perfectly
18
Q

True or false: Under FRCP 14(a), impleading is allowed in any state, no matter the rule.

A

False. Parties can only implead if they have a right to contribution from them. Ex: if Erkins arose in a state that does not allow contribution among tortfeasors, Case would not be able to implead the other contractors, since it would have no right to pass on part of its liability.

How well did you know this?
1
Not at all
2
3
4
5
Perfectly
19
Q

Quinn is arrested by a red-headed police officer and sues Jones, a red-headed police officer who was on duty at the time. Quinn claims that Jones battered him during the arrest. Jones believes it was Smith who arrested Quin rather than him, so he impleads Smith.

Is impleader proper?

A

No.

If Jones made the arrest, he is liable to Quinn. If Smith did it, he is liable to Quinn. But there is no theory on which Smith would be liable to Jones.

Here, Jones is offering a different target for Quinn = “It wasn’t me, it was him!” That is not the rule of impleader under FRCP 14(a). Rather than implead Smith, Jones should deny that he arrested Quinn in his answer to Quinn’s complaint. Quinn could then move to amend to add Smith as an additional defendant under FRCP 20(a)(2) if he is unsure who arrested him.

How well did you know this?
1
Not at all
2
3
4
5
Perfectly
20
Q

Masters is injured in an accident due to the negligence of two other drives, Grainger and Ribeiro. Masters sues Grainger for his injuries and Grainger impleads Ribeiro under FRCP 14 for contribution.

Is this impleader proper?

A

Yes. A party can be impleaded if he is liable directly to P as long as he may be liable to the main D for contribution or indemnification as well.

Ribeiro would be liable directly to Masters for his injuries. Masters could have sued him alone or with Grainger as a joint tortfeasor. But if Masters chooses to sue Grainger alone and recovers his damages from Grainger, Ribeiro would be liable to Grainger for contribution if Grainger paid the full judgment and joint and several liability applies.

Grainger has paid a liability that they share and has a right to get part of it back from Ribeiro.

How well did you know this?
1
Not at all
2
3
4
5
Perfectly
21
Q

If a defendant meets the standard for impleading a third party, but the third party is not subject to PJ in the state where the action is pending… can the third-party be impleaded?

A

No. Third-party defendants have due process rights where they cannot be sued in a court that lacks jurisdiction over them.

Whether under FRCP 20 or FRCP 14(a), defendants may raise an objection to PJ

How well did you know this?
1
Not at all
2
3
4
5
Perfectly
22
Q

Wu buys a building lot from Caudel and Polansky. He learns that the water table is too high to build a septic system on it and consequently no building can be constructed there. Claiming misrepresentation, Wu sues Caudel to rescind the sale.

Could the case proceed with just Caudel?

A

A court would probably require Wu to sue Polansky as well as Caudel. The court could not rescind the sale without having Polansky before it, since the court cannot order Polansky to return his part of the purchase price or rescind the sale unless he is before the court.

The court could not give Wu the remedy sought without both Caudel and Polansky.

How well did you know this?
1
Not at all
2
3
4
5
Perfectly
23
Q

City Realty Group rents the 20th floor of a skyscraper to Stellar Corp., which then sublets to Taylor publishing. The lease and the sublease both include a requirement that a reasonable access to utilities will be be provided to the lessee.

After moving in, Taylor refuses to pay rent to Stella because the power supply is inadequate. Stellar claims that City Realty has caused the problem by refusing to upgrade the power supply to accommodate Taylor’s needs.

Stellar sues to collect the rent, but Taylor claims it doesn’t owe rent because of the owner’s unreasonable refusal to accommodate its electrical needs.

Who should be required to join the case?

A

Has there been a breach of sublease? We don’t know if City Realty isn’t added as a party to the claim. Thus, City Realty should be added so the court can decide whether it has a duty to upgrade the power or whether Stellar agreed to a provision that they couldn’t fulfill. Affording relief may also require an order to City Realty to comply.

How well did you know this?
1
Not at all
2
3
4
5
Perfectly
24
Q

Mr. Moreno sues Merrill for damages arising from a business transaction that Merrill entered into on behalf of his wife and himself. Moreno discovers that all of Merrill’s assets are held as “community property” with his wife.

Would adding Merrill’s wife be appropriate?

A

It will be impossible to collect a judgment without getting a joint judgment against Merrill and his wife. If Moreno recovers damages, the court cannot order execution on Mr. Merrill’s community property assets without entry of a judgment against Mrs. Merrill as well.

How well did you know this?
1
Not at all
2
3
4
5
Perfectly
25
Q

Window Glass Cutters Union sues the employer of its members, claiming that the employer violated the collective bargaining agreement by failing to assign certain work to its members.

The employer’s position is that the employees actually perform work covered by a different collective bargaining agreement, with the US Glass and Ceramic Workers.

The employer moves to join the US Glass and Ceramics workers under FRCP 19(a). Should the second union be joined? If so, under which subsection of the rule?

A

Yes. Rule 19(a)(1)(B)(ii) covers the case.

If the action proceeds and the court holds that work must be assigned to workers covered by the Window Glass Cutters Union, it will order the employer to do so.

Very likely, the employer will then be sued by the US Glass and Ceramic workers, claiming the work should have been assigned to its members instead.

The United Glass union would not be bound by that decision (since it was not a party to the first action), so it could litigate the question again in a later action. In that second action, the court might find US Glass workers entitled to the work, so that employer would be stuck between two judgments.

How well did you know this?
1
Not at all
2
3
4
5
Perfectly
26
Q

True or false: In litigation, the reasons parties offer in support of a motion may differ from their motives for making the arguments?

A

True.

Ex: Yost made his motion to dismiss even if he was not particularly concerned about incurring an inconsistent obligation to INA. He may have used Rule 19 as a forum-shopping tool. He may have brought the motion because he and INA preferred state court.

Yost was able to show that INA was indispensable, thus the case would be dismissed from federal court for lack of jurisdiction, leaving Torrington to sue in state court instead.

How well did you know this?
1
Not at all
2
3
4
5
Perfectly
27
Q

Cardena (from Michigan) hires Kelleher Plumbing to do the plumbing work on his new house and Nashoba Electric Company to do the wiring. Cardena is unhappy with both jobs and sues Nashoba (from Wisconsin) in federal court. Nashoba moves to join Kelleher (from Michigan) in the federal action under Rule 19.

How should the court rule?

A

The court should deny the motion because nothing in Rule 19 makes Kelleher a person to be joined if feasible.

Rule 19 (a)(1)(A) does not require joinder, b/c Cardena can recover fully from Nashoba for any defects in its work without litigating his claim against Kelleher in the federal suit.

Rule 19(a)(1)(B) is not implicated b/c Kelleher won’t be affected in any way by this suit between Cardena and Nashoba about Nashoba’s unrelated work.

Cardena here has two claims based on separate disputes with different contractors on a single job.

How well did you know this?
1
Not at all
2
3
4
5
Perfectly
28
Q

Cardena (from Michigan) hires Kelleher Plumbing (from Michigan) to do the plumbing work on his new house and Nashoba Electric Company (from Wisconsin) to do the wiring.

The house burned down while Kelleher and Nashoba were both working on it. Cardena sues Nashoba in federal court for the damage to the house, claiming that its negligence cause the fire. Nashoba moves to dismiss for failure to join Kelleher, arguing that negligence of Kelleher’s plumbers may have caused the fire instead, or the plumber’s negligence have contributed to the fire along with Nashoba’s, so it must also be made a defendant.

What should the court decide?

A

The court should not order Kelleher joined under either Rule 19 or Rule 20.

Courts uniformly hold that Rule 19 does not require joinder of other tortfeasors in cases like this. The plaintiff has chosen her defendant. If she proves its liability, she will recover. If not, she will lose. But nothing requires that another potentially liable person be joined to effectively litigate the claim against the first tortfeasor.

How well did you know this?
1
Not at all
2
3
4
5
Perfectly
29
Q

True or false: Rule 20(a) addresses when a party should be ordered joined by a court

A

False. Rule 20(a) addresses when parties may be made parties at the plaintiff’s option, not when they should be ordered joined by the court.

How well did you know this?
1
Not at all
2
3
4
5
Perfectly
30
Q

What are the three “hoops” a court must address under Rule 24(a)?

A

Rule 24: Intervening Parties

1) Applicant’s interest
2) likely impairment of that interest if the action goes forward without the applicant
3) adequacy of representation by OG parties

How well did you know this?
1
Not at all
2
3
4
5
Perfectly
31
Q

A utility sued its gas supplier claiming that the supplier was overcharging under the contract. Customers of the utility moved to intervene, claiming an interest in the case b/c their rates would rise if the supplier won, leading to an increased cost to the utility for gas.

Is the interest sufficient under 24(a)(2)?

A

No. Although the customers may suffer an indirect economic impact from the case, they had no direct interest in the contract at issue in the case.

How well did you know this?
1
Not at all
2
3
4
5
Perfectly
32
Q

The Sierra Club sued the Secretary of Agriculture, claiming clear-cutting in national forests in Texas violated federal laws. Two trade associations, representing most of the purchasers of timber from the Texas national forest sought to intervene as defendants, to argue that clear-cutting was authorized by federal law.

Is the interest sufficient under 24(a)2)?

A

Yes. The timber association has a “legally protectable property interest” in the dispute because the members’ current and future contracts to harvest timber in the forest would be affected.

How well did you know this?
1
Not at all
2
3
4
5
Perfectly
33
Q

What are some of the similarities between Rule 19(a) and Rule 24(a)?

A

Both rules refer to a person who claims an interest relating to the subject of the action and is so situation “that disposing of the action in the person’s absence may… as a practical matter impair or impede the person’s ability to protect the interest.”

If a person is a required party under Rule 19(a)(1)(B)(i) he is also entitled to intervene under Rule 24(a)(2).

How well did you know this?
1
Not at all
2
3
4
5
Perfectly
34
Q

What are some of the differences between Rule 19(a) and Rule 24(a)?

A

Who seeks joinder. Typically defendants move for joinder of a party under Rule 19 while the absentee moves to intervene under Rule 24.

Interest –> not the same purpose for one or the other. There are times when a petitioner should be allowed to intervene under Rule 24 that are not necessarily limited to those situations when the trial court should compel them under Rule 19. Some applicants aren’t necessary parties.

How well did you know this?
1
Not at all
2
3
4
5
Perfectly
35
Q

If a year after the suits in Grutter were filed, the individuals and advocacy groups who originally intervened in Grutter were granted the right to intervene. Twenty three more potential applicants to the University, living in New York, learn of the action from an article about the intervention decision published in the NY Times.

They quickly file an application to intervene as defendants under Rule 24, to argue in support of the University’s affirmative action programs.

How should a court rule?

A

The application should be denied, since their interest are adequately represented by those already parties to the action.

Unless this other group of applicants is somehow distinguishable from the first group, int terms of their interests and arguments in support of the University’s affirmative action program, the court is likely to avoid an expansion of the case by denying intervention on the grounds that the later applicants’ interests are adequately represented by the original intervenors.

How well did you know this?
1
Not at all
2
3
4
5
Perfectly
36
Q

True or false: application to intervene under Rule 24 has strict timelines because the rule states that applications must be “timely”

A

False. Timeliness under Rule 24 is not measure rigidly from commencement of the action. It will be judged practically based on whether the intervenors acted promptly after becoming aware of the action.

How well did you know this?
1
Not at all
2
3
4
5
Perfectly
37
Q

True or false: Intervention under rule 24(a) is an all-or-nothing proposition

A

False. A party might be allowed to intervene for limited purposes or only on certain claims in the case.

+ Stringfellow: neighborhood group could intervene on certain claims but not others

+ Pansy: limited purpose –> efforts of a news organization to obtain info filed in court under a confidentiality order

+ Public Citizen: limited purpose –> citizen’s group sought to seek access to discovery documents produced in litigation

+ interested non-parties can file amicus briefs (friend of the court) in support of one party’s position

How well did you know this?
1
Not at all
2
3
4
5
Perfectly
38
Q

True or false: Denial of an application to intervene as a right of final decision must wait until the of the case.

A

False: Denial of an application to intervene as a right is a final decision that is immediately appealable. The intervenor may appeal right away, arguing that she should be allowed to participate.

How well did you know this?
1
Not at all
2
3
4
5
Perfectly
39
Q

True or false: Once an intervention is granted on appeal, the opposing party can immediately appeal

A

False. A party who objects to the intervenor’s presence (via grant of appeal) must wait until the end of the case to appeal.

As a practical matter, if the intervention is granted on appeal, it is effectively unreviewable. If you want to win this battle, you must win it in front of the trial judge.

How well did you know this?
1
Not at all
2
3
4
5
Perfectly
40
Q

A Nevada insurer holds $100,000 in proceeds of a policy, claimed by four claimants, three from IA and one from Nevada.

Could this be brought in an interpleader case under FRCP 22 or §1335?

A

Statutory interpleader under §1335(a) is proper here:

1) has possession of money or property worth more than $500 - YES
2) There are two or more diverse claimants to the money or property - IA and NV - YES
3) P has deposited the money or property in the court or given a bond for compliance with the court’s eventual order for payment of the money or property - YES

Rule 22 interpleader NOT proper here:

1) Claimants must be subject to PJ under traditional analysis, under FRCP 4(k)(1)(A) - minimum contacts?
2) SMJ must be based on traditional rules of diversity under §1332(a) or federal question jurisdiction under §1331 - N/A because it’s a contracts claim and parties are from the same state (lack of complete diversity)

How well did you know this?
1
Not at all
2
3
4
5
Perfectly
41
Q

A KY county holds a $50,000 reward for apprehension of an armed robber. Four AR citizens claim that their information led to the arrest.

Could this be brought in an interpleader case under FRCP 22 or §1335?

A

Statutory interpleader under §1335(a) not proper:

1) has possession of money or property worth more than $500 - YES
2) There are two or more diverse claimants to the money or property - NO - everyone is from AR
3) P has deposited the money or property in the court or given a bond for compliance with the court’s eventual order for payment of the money or property - YES

Rule 22 interpleader NOT proper here:
Amount in Controversy does not exceed $75K and this is not a federal question of law under §1331

How well did you know this?
1
Not at all
2
3
4
5
Perfectly
42
Q

An art gallery, incorporated in IL with its principal place of business there, holds a painting valued at $125,000 that is claimed by four heirs of the decedent. All four heirs live in OK.

Could this be brought in an interpleader case under FRCP 22 or §1335?

A

Statutory interpleader under §1335(a) not proper:

1) has possession of money or property worth more than $500 - YES
2) There are two or more diverse claimants to the money or property - NO - all claimants are from OK
3) P has deposited the money or property in the court or given a bond for compliance with the court’s eventual order for payment of the money or property - YES

Rule 22 interpleader proper:

1) Claimants must be subject to PJ under traditional analysis, under FRCP 4(k)(1)(A)
2) SMJ must be based on traditional rules of diversity under §1332(a): P is from IL and D from OK - complete diversity met AND AIC is over $75K

How well did you know this?
1
Not at all
2
3
4
5
Perfectly
43
Q

What is the main difference between FRCP 13 and 14?

A

FRCP 13(g) relates to crossclaims against another D to recover for an independent loss. FRCP 14 is a claim to pass on liability from one D to another.

How well did you know this?
1
Not at all
2
3
4
5
Perfectly
44
Q

True or false: Many FRCP 14 claims are for contribution by co-defendants in a tort case.

A

True. These types of claims are only allowed if the law of the forum state allows contributions among joint tortfeasors.

45
Q

True or false: FRCP 14 can be used just to bring in another possible Defendant who may be liable to the Plaintiff.

A

False.
FRCP 14 CANNOT be used just to bring in another possible Defendant who may be liable to the Plaintiff. The original Defendant (TPP) must have a claim for reimbursement from TPD.

46
Q

What is the main difference between FRCP 19 and FRCP 24?

A

Under Rule 24: As a practical matter impair or impede the movant’s ability to protect its interest, **unless existing parties adequately represent that interest **

If someone else in the lawsuit is already protecting the interest of a non-party, then they may not be allowed to intervene.

47
Q

True or false: In intervention, a potential intervenor needs to prove res judicata impairment

A

False. Potential intervenors need not prove res judicata impairment, stare decisis effect is enough.

48
Q

C (resident of S. Dakota) is arrested by E and R (both from South Dakota), two police officers. He claims the officers assaulted him during the arrest. C sues the officers under 42 U.S.C. §1983, alleging they violated his federal constitutional rights to be free of unreasonable seizures. He also asserted a state law claim for battery against both E and R.

C properly joined both officers as codefendants under Rule 20(a)(2) because he seeks relief arising from the same arrest, and there will be a common question as to who assaulted him.

Does the federal court have SMJ over C’s claims?

A

The federal court has SMJ as to C’s federal claims, since they arise under federal law.

However, C’s battery claims are based on state law against E and R are not within the federal court’s SMJ. All three parties are for the same state and there’s a lack of complete diversity (no P’s can share the same citizenship as any D’s). There is no independent basis for SMJ on the battery claim alone and a court would dismiss for lack of SMJ.

BUT can C add the state law claims once he’s established SMJ on the federal question of law? Supplemental jurisdiction says yes –> Pendent jurisdiction

49
Q

True or false: If a P asserts one proper federal claim, the court may hear related state law claims if they arise out of the same “common nucleus of operative fact”

A

True. Gibbs holds this to be true. The standard focuses on the facts involved in a dispute, not on legal theories. In Gibbs, both the state and federal claims were based on the same underlying event.

50
Q

True or false: In deciding pendent jurisdiction, a trial court may not decline jurisdiction.

A

False. A trial court need not always exercise pendent jurisdiction even if they have the power to do so. Reasons can be related to judicial economy, convenience, and fairness to litigants.

51
Q

What are some reasons why a federal court may decline jurisdiction the doctrine of pendency?

A

1) Federal claim drops out early (leaving court with a state claim only) –> however federal court may retain jurisdiction if they so choose if the federal claim is dropped later in the case/significant pre-trial litigation has occurred.
2) State issue is predominate –> “vast majority” is state claims only
3) If the state law claims present novel or complex issues of state law –> “surer footed reading” of state law in state court
4) Likelihood of jury confusion

52
Q

Arch Technologies (incorporated in OR and principal place of biz in CA) sues Sullivan Castings Corp. (incorporated in DE and principal place of biz in OR) in federal court under a federal statute, barring deceptive commercial practices in interstate commerce.

Arch claims losses of $500K and also claims that Sullivan’s conduct constitutes an unfair business practice under an Oregon statute and seeks punitive damages under the state law claim. Oregon courts have never decided whether punitive damages may be awarded under its business practices statutes.

The federal judge wants to decline jurisdiction over the unfair business practices claim, since the government law is unclear and the Oregon court can provide a “surer footed” reading of the statute.

What should the judges do?

A

The judges should dismiss the state law claim, retaining jurisdiction over the federal claim.

The court cannot remand the state law claim to Oregon state court, nor can they remand the case to the Oregon state court. The judge can’t dismiss the entire case, even if it would be more efficient to dismiss the entire case.

If the judge does dismiss the state law claim, P may voluntarily dismiss the federal suit and breing the fed claim and state claim together in state court (that’s allowed!). P can also keep her federal claim in federal court and file her state claim in state court.

53
Q

Under what circumstances may a federal court hear and decide a state law claim arising between citizens of the same state?

A

Cases where the claim arises from a question of federal law §1331

54
Q

True or false: In the Kroger case, would it be constitutionally impermissible for the court to hear Kroger’s claim

A

False. Even though the Kroger court concluded that exercising jurisdiction over Kroger’s claim was inconsistent with the diversity statute, Gibbs “delineated the constitutional limits of federal judicial power.”

Gibbs held that a federal court with original jurisdiction may hear all other claims in the case that arise from the same nucleus of operative fact, federal courts have a constitutional power her Kroger’s claim.

Then the court turned to §1332 diversity jurisdiction and determined that they could not hear the case.

55
Q

Adair (from CA) sues the US for injuries suffered in a plane crash at a California airport. Her suit is brought under the Federal Tort Claims act, which authorizes federal courts to hear tort claims against the US. She argues that negligence of a federal air traffic controller led to the crash.

She also sues Vasquez (from CA) for the same injuries, claiming that Vasquez, flying a private plane, got in the way as he was landing.

Considering §1367 (a) and (b), would a federal district court have jurisdiction over Adair’s claim against Vasquez?

A

Yes because the court had original jurisdiction over the claim against the U.S. under §1346(a), which authorizes jurisdiction of tort claims against the US.

Under §1367(a), the court has jurisdiction over Adair’s claim against Vasquez, which arises from the same airplane crash. The exception to §1367(b) does not apply because the federal court’s original jurisdiction is not based on §1332 (but a question of federal law).

56
Q

Crowley (CO) sues Picard (IA) for $200,000, claiming that Picard — a broker in a real estate deal — misrepresented septic problems with the property, inducing Crowley to purchase it.

Picard impleads Banchevsky (CO), the seller, for indemnification, arguing that if she misrepresented the septic problems to Crowley, it was because Bachevsky had given her the wrong information about thej.

Does the court have jurisdiction over Picard’s claim against Banchevsky?

A

Yes, it does. Supplemental jurisdiction is unnecessary here, because it is a claim between an Iowa citizen and a Colorado citizen (complete diversity) for more than $75K (AIC) which is valid under §1332.

Picard seeks reimbursement of the entire amount Crawley seeks from her, that’s what indemnification means. As long as Crowley’s claim is arguably more than $75K, Picard’s claim against Banchevsky satisfies the standards of original jurisdiction in federal court.

No need to consider whether supplemental jurisdiction applies.

57
Q

Crowley (CO) sues Picard (IA) for $200,000, claiming that Picard — a broker in a real estate deal — misrepresented septic problems with the property, inducing Crowley to purchase it.

Picard impleads Banchevsky (IA), the seller, for indemnification, arguing that if she misrepresented the septic problems to Crowley, it was because Bachevsky had given her the wrong information about thej.

Does the court have jurisdiction over Picard’s claim against Banchevsky?

A

If Banchevsky is from IA, there is no diversity between her and Picard, so we must look for another basis for jurisdiction over Picard’s impleader claim.

The main suit is based on diversity, not a federal question, but supplemental jurisdiction applies to diversity cases as well as federal question cases.

The court has original jurisdiction over Crowley and Picard’s case based on diversity. The court has supplemental jurisdiction over the impleader claim if it arises out of the same case or controversy as the main case under §1367(a). Here, it does since since Picard’s indemnification claim against Bachevsky arises from the same sale of property as the main claim.

B.C. original jurisdiction is based on diversity, we need to consider whether §1367(b) excepts this claim for grant of supplemental jurisdiction. It does not, since the original P is not asserting this claim.

58
Q

True or false: For the court to have “original jurisdiction” over ANY initial claim in a diversity case, there must be complete diversity.

A

True. In order for a federal court to invoke supplemental jurisdiction under Gibbs, it must first have original jurisdiction over at least one claim in the action.

59
Q

Does a court have to look at all parties when it analyzes both complete diversity requirements and AIC requirements?

A

A court must look at ALL parties when analyzing complete diversity requirements, but separately when analyzing AIC.

A court may base original jurisdiction on a single claim if the case arises under federal law, but must consider all of the original plaintiff’s claims in a diversity case.

60
Q

True or false: Under FRCP 20, the joinder of parties is permissive when P or D assert claims that
“arises out of the same transaction, occurrence, or series of transactions or occurrences (STOO); or
If their claims involve any question of law or fact common to all P’s”

A

False

Under FRCP P or D can sue together if they assert claims that “arises out of the same transaction, occurrence, or series of transactions or occurrences (STOO); and If their claims involve any question of law or fact common to all P’s” (requires both)

61
Q

How is a counterclaim different from a denial or affirmative defense under Rule 13?

A

Party asserting counterclaim is seeking their own relief?

62
Q

Do 13(a) claims meet supplemental test for related claims under §1367?

A

YES but 13(b) counterclaims nee their own SMJ analysis.

63
Q

What are the exceptions to compulsory counterclaims?

A

1) Subject to another pending action
2) Juris over main claim not PJ
3) arises after answer

64
Q

True or false: Crossclaims are always permissive?

A

True, unlike Rule 13(a) Crossclaims are always permissive

65
Q

True or false: Rule 14 allows third-party D to assert defenses that may defeat the main claim but not defenses to the third-party claim

A

False: Rule 14 Allows third-party D to assert defenses that may defeat the main claim as well as defenses to the third-party claim

66
Q

True or false: D’s can add parties under Rule 20

A

False. D’s can join parties in counterclaims or crossclaims under Rule 13(h)

67
Q

What are the primary similarities and differences between Rules 19 and 24?

A

Rule 19: Requiring an absentee to be made party to a case
Rule 24: Intervention: Who’s allowed?
When nonparties ask to participate in a case where they are not named → practical impact on their interests

Similarities in language:
+ Claims an interest relating to the subject of the action
+ Impair or impede the person’s ability to protect the interest

68
Q

True or false: §1291 limits appeals to final decisions and motions to intervene

A

Part true, part false

§1291 Limits appeal to final decisions, however dismissal of a motion to intervene is immediately appealable

69
Q

Is an order granting intervention immediately appealable?

A

No, it’s effectively unappealable harmless error.

70
Q

*Plaintiff, a citizen of Louisiana, filed an action in federal district court in New Orleans. His complaint contained a federal civil rights claim alleging that he had been the victim of unlawful sexual harassment and a breach of contract claim against his employer, also a citizen of Louisiana. The breach of contract claim alleged that Defendant had failed to pay Plaintiff overtime as required by his employment contract.

Are all of P’s claims against D joinable?

A

Yes, because his right of joinder is unlimited.

Here we have a plaintiff asserting two claims against one defendant. The relevant standard is contained in FRCP 18. Under Rule 18, a party can join as many claims as he or she has against that adverse party, regardless of whether the claims are related or unrelated. Keep in mind that the court would still need subject matter jurisdiction to adjudicate the controversy, but that is not the question at issue. Rule 18 deals only with pleading. A claim properly joined as a matter of pleading need not be proceeded with together with the other claim if fairness or convenience justifies separate treatment. Also, Rule 18 does not purport to deal with questions of jurisdiction or venue which may arise with respect to claims properly joined as a matter of pleading.

71
Q

Plaintiff, a citizen of Louisiana, filed an action in federal district court in New Orleans. His complaint contained a federal civil rights claim alleging that he had been the victim of unlawful sexual harassment and a breach of contract claim against his employer, also a citizen of Louisiana. The breach of contract claim alleged that Defendant had failed to pay Plaintiff overtime as required by his employment contract.

Plaintiff also asserted a tort claim for intentional infliction of emotional distress against his manager, a citizen of Louisiana, alleging that it was her intentional acts of sexual harassment that caused him to suffer severe emotional distress. In addition to filing an answer, Defendant employer asserted a breach of contract claim against Plaintiff and an indemnity claim against the manager. In its breach of contract claim, Defendant alleged that Plaintiff had not honored his contractual commitment to attend a variety of off-site conferences.

Is D’s claim against the manager joinable?

A

Yes, because this claim arises out of the transaction that gave rise to Plaintiff’s claim against Defendant.

Defendant has filed a cross-claim against the manager. The rule governing the joinder of cross-claims in federal court is FRCP 13(g). Like the rule regarding joinability of third-party claims, and unlike the rules governing the joinability of counterclaims, the rule governing joinability of cross-claims is limited. Rule 13(g) only permits the joinability of cross-claims that arise out of the transaction or occurrence that gave rise to the plaintiff’s claims or to a counterclaim. Defendant’s cross-claim against the manager for indemnity in connection with any liability it may incur as a result of Plaintiff’s claim against it clearly arises out of the occurrences that gave rise to the plaintiff’s claim against Defendant.

72
Q

*Plaintiff, a citizen of Louisiana, filed an action in federal district court in New Orleans. His complaint contained a federal civil rights claim alleging that he had been the victim of unlawful sexual harassment and a breach of contract claim against his employer, also a citizen of Louisiana. The breach of contract claim alleged that Defendant had failed to pay Plaintiff overtime as required by his employment contract. Plaintiff also asserted a tort claim for intentional infliction of emotional distress against his manager, a citizen of Louisiana, alleging that it was her intentional acts of sexual harassment that caused him to suffer severe emotional distress.

Is Plaintiff’s claim against the manager joinable?

A

Yes, because his claim against the manager arises out of the same occurrences that gave rise to his claim against his employer and both claims contain a common question of fact.

Plaintiff is seeking to bring claims against more than one defendant in this suit, which automatically makes this a joinder of parties problem, not a joinder of issues problem. The rule governing joinder of parties in federal court is FRCP 20. Under Rule 20(a)(2), a plaintiff can assert claims against more than one defendant as long as both of two requirements are met. First, these claims must arise out of the same transaction, occurrence, or series of transactions or occurrences. Second, the claims against both of the defendants must contain at least one common question of law or fact.

Here, the claims against both Defendant and the manager arise out of the latter’s alleged acts of harassment. So the first requirement of Rule 20 is met. Second, both of these claims require resolution of, among other things, the factual question of what the manager did to Plaintiff. Consequently, both requirements of Rule 20(a)(2) have been met.

73
Q

*Plaintiff, a citizen of Louisiana, filed an action in federal district court in New Orleans. His complaint contained a federal civil rights claim alleging that he had been the victim of unlawful sexual harassment and a breach of contract claim against his employer, also a citizen of Louisiana. The breach of contract claim alleged that Defendant had failed to pay Plaintiff overtime as required by his employment contract. Plaintiff also asserted a tort claim for intentional infliction of emotional distress against his manager, a citizen of Louisiana, alleging that it was her intentional acts of sexual harassment that caused him to suffer severe emotional distress. In addition to filing an answer, Defendant employer asserted a breach of contract claim against Plaintiff and an indemnity claim against the manager. In its breach of contract claim, Defendant alleged that Plaintiff had not honored his contractual commitment to attend a variety of off-site conferences.

Is Defendant’s claim against Plaintiff joinable?

A

Yes, because the right of joinder of counterclaims is unlimited.

Defendant’s claim against Plaintiff is a counterclaim and the rules governing joinder of counterclaims in federal court are found in FRCP 13(a) and (b). Rule 13(a) defines a compulsory counterclaim and 13(b) defines permissive counterclaims. Whether or not a joinable counterclaim is deemed to be compulsory is determined by assessing whether that counterclaim meets the requirements of 13(a) or (b). But that is not the issue here. The issue here is not whether the defendant was required to assert this counterclaim, but whether he is permitted to do so, i.e., whether the counterclaim is joinable. The combination of these two subsections of Rule 13 clearly means that all counterclaims are joinable.

Rule 13 is only a rule of pleading and does not purport to override or eliminate the jurisdictional and venue requirements under the U.S. Code. Thus, the question asks only whether the claim is joinable, not whether the court will ultimately be able to adjudicate the claim based on subject matter jurisdiction.

74
Q

Plaintiff, a citizen of Louisiana, filed an action in federal district court in New Orleans. His complaint contained a federal civil rights claim alleging that he had been the victim of unlawful sexual harassment and a breach of contract claim against his employer, also a citizen of Louisiana. The breach of contract claim alleged that Defendant had failed to pay Plaintiff overtime as required by his employment contract. Plaintiff also asserted a tort claim for intentional infliction of emotional distress against his manager, a citizen of Louisiana, alleging that it was her intentional acts of sexual harassment that caused him to suffer severe emotional distress.

In addition to filing an answer, Defendant employer asserted a breach of contract claim against Plaintiff and an indemnity claim against the manager. In its breach of contract claim, Defendant alleged that Plaintiff had not honored his contractual commitment to attend a variety of off-site conferences. Finally, the manager responded to all of this by filing a third-party complaint against her insurance company, Insurance Co., a citizen of Connecticut. In this complaint, the manager asserted both a breach of contract claim alleging that Insurance Co. had refused to pay a covered claim under her homeowner’s policy and a state law claim for indemnity (under her professional liability policy) in connection with any liability she would incur as a result of Plaintiff’s cause of action against her.

Are the manager’s claims against Insurance Co. joinable?

A

Yes, as to both claims

The manager has filed a third-party complaint against Insurance Co. and this complaint contains two causes of action. The rule governing third-party complaints in federal court is Rule 14. Under Rule 14(a), third-party claims are joinable, but ONLY if they are for indemnity or contribution. Thus, unlike Rule 18 as to multiple claims by one party against one opposing party and Rule 13 as to counterclaims, the rule of joinder of third-party claims is limited. So the indemnity claim is joinable under Rule 14(a). And while it is true that the other non-indemnity claim for breach of contract is not joinable under Rule 14(a), it is joinable under Rule 18 once the indemnity claim is joined because Rule 18 permits unlimited joinder of additional claims to a joinable claim or third-party claim. Thus, but for the indemnity claim (joinable under Rule 14) the insurance company would not be a party to the action at all.

While it is true that the other non-indemnity claim is not joinable under Rule 14(a), it is joinable under Rule 18, which permits unlimited joinder of additional claims to a joinable claim or third-party claim.

75
Q

Plaintiff, a citizen of State P, ate dinner one night at a restaurant operated in State P by Defendant, a citizen of State V. After eating dinner, Plaintiff became ill. It was determined that the produce at the restaurant was contaminated. Defendant had bought produce from a third party, a citizen of State P who is in the business of delivering produce to restaurants. This seller had bought the produce from a farmer who is a citizen of State V.

After months of hospitalization, Plaintiff commenced a $350,000 tort action in State P federal district court against Defendant. Defendant then impleaded the produce seller and the farmer as third-party defendants, under an indemnification theory. Plaintiff then sought to amend the complaint to add similar claims against the seller and farmer.

Seller moved to dismiss Plaintiff’s claim against him for lack of subject matter jurisdiction. Farmer moved to dismiss Defendant’s third-party claim against him on lack of subject matter jurisdiction.

How should the court rule on each motion?

A

The court should grant seller’s motion but not grant farmer’s, because Plaintiff’s suit against seller does not fall within the court’s supplemental jurisdiction but Defendant’s suit against farmer does.

Pursuant to FRCP 14, seller is a third-party defendant. A defending party is permitted to bring into the lawsuit a nonparty who is or may be liable to the defending party for all or part of the original plaintiff’s claim against the defending party. The additional party is called the third-party defendant and the defending party is called the third-party plaintiff. This impleader action does not need an independent basis for subject matter jurisdiction; so long as the original action satisfies the requirements, the courts will exercise supplemental jurisdiction over the third-party claims.

Since the claim by the original defendant against a third-party defendant falls within the court’s supplemental jurisdiction, Defendant’s third-party claim against the farmer will not be dismissed, even though the Defendant and the farmer are both from State V.

However, a claim by the original plaintiff against the third-party defendant does not fall within the court’s supplemental jurisdiction, so it must have independent basis for subject matter jurisdiction. Here, Plaintiff and seller are both citizens of State P, so there is no diversity of citizenship. Also, under the supplemental jurisdiction statute, the federal court cannot exercise supplemental jurisdiction over a plaintiff’s claims against persons made parties under Rule 14 when doing so would destroy diversity jurisdiction. The court must dismiss Plaintiff’s claim against seller.

76
Q

Plaintiff, a lifelong resident of Atlanta, Georgia, brought suit in federal district court in Atlanta against a publisher, Books, Inc., alleging copyright infringement under the federal copyright statute, for which she sought $100,000 in damages. Books, Inc. is incorporated under the laws of Delaware and has its principal place of business in New York City. Plaintiff also filed a claim against Books, Inc. alleging that it had breached its agreement with her to publish her book upon receipt of a manuscript. She sought $60,000 in damages in connection with that claim. In that same suit, Plaintiff also filed a $105,000 tort claim against the president of Books, Inc., a citizen of New York, alleging that he had intentionally inflicted emotional distress upon her by sending emails to dozens of other publishers denouncing her as a horrible writer and a fraud and explaining that those were the reasons for his company’s refusal to publish her book. In this same action, Books, Inc. filed a claim against President seeking indemnity for any liability it might accrue in connection with Plaintiff’s copyright infringement claim against it.

Is Books, Inc.’s claim against President joinable?

A

Yes, because it is an indemnity claim.

This is a cross-claim for indemnity. The Rule governing joinder of cross-claims is 13(g). Unlike Rule 18 and Rule 13 governing joinder of multiple claims by a single plaintiff against a single defendant and joinder of counterclaims, respectively, Rule 13(g) does not provide an unlimited right of joinder. It only permits joinder of cross-claims that are transactionally related to the original complaint or counterclaim.

77
Q

*Plaintiff, a lifelong resident of Atlanta, Georgia, brought suit in federal district court in Atlanta against a publisher, Books, Inc., alleging copyright infringement under the federal copyright statute, for which she sought $100,000 in damages. Books, Inc. is incorporated under the laws of Delaware and has its principal place of business in New York City. Plaintiff also filed a claim against Books, Inc. alleging that it had breached its agreement with her to publish her book upon receipt of a manuscript. She sought $60,000 in damages in connection with that claim. In that same suit, Plaintiff also filed a $105,000 tort claim against the president of Books, Inc., a citizen of New York, alleging that he had intentionally inflicted emotional distress upon her by sending emails to dozens of other publishers denouncing her as a horrible writer and a fraud and explaining that those were the reasons for his company’s refusal to publish her book.

Is P’s claim against Books, Inc. joinable?

A

Yes, because there is no limit to joinability of such claims.

The plaintiff is seeking to join two claims against one defendant. Under Rule 18, there is no limit to the number of claims a party can join against another party as long as one of the claims is joinable.

78
Q

*Plaintiff, a lifelong resident of Atlanta, Georgia, brought suit in federal district court in Atlanta against a publisher, Books, Inc., alleging copyright infringement under the federal copyright statute, for which she sought $100,000 in damages. Books, Inc. is incorporated under the laws of Delaware and has its principal place of business in New York City. Plaintiff also filed a claim against Books, Inc. alleging that it had breached its agreement with her to publish her book upon receipt of a manuscript. She sought $60,000 in damages in connection with that claim. In that same suit, Plaintiff also filed a $105,000 tort claim against the president of Books, Inc., a citizen of New York, alleging that he had intentionally inflicted emotional distress upon her by sending emails to dozens of other publishers denouncing her as a horrible writer and a fraud and explaining that those were the reasons for his company’s refusal to publish her book. In this same action, Books, Inc. filed a claim against President seeking indemnity for any liability it might accrue in connection with Plaintiff’s copyright infringement claim against it. In response, President filed a breach of contract claim against Books, Inc. alleging that Books, Inc. had improperly withheld two weeks of salary from him totaling $65,000.

Is President’s breach of contract claim against Books, Inc., joinable?

A

Yes, because there is no limit to joinability of this type of claim.

Although this is a claim between co-defendants, it is not a cross-claim, but a counterclaim to Books, Inc.’s cross-claim against President. Consequently, the correct Rule governing joinability is Rule 13(a) and (b) and not Rule 13(g). There is no limit on joinability of counterclaims and so this claim is joinable. Whether or not the court has subject matter jurisdiction is another matter, but that is not being tested in this question. Rule 13 is only a rule of pleading and does not purport to override or eliminate the jurisdictional and venue requirements under the U.S. Code. Thus, the question asks only whether the claim is joinable, not whether the court will ultimately be able to adjudicate the claim based on subject matter jurisdiction.

79
Q

Plaintiff, a lifelong resident of Atlanta, Georgia, brought suit in federal district court in Atlanta against a publisher, Books, Inc., alleging copyright infringement under the federal copyright statute, for which she sought $100,000 in damages. Books, Inc. is incorporated under the laws of Delaware and has its principal place of business in New York City. Plaintiff also filed a claim against Books, Inc. alleging that it had breached its agreement with her to publish her book upon receipt of a manuscript. She sought $60,000 in damages in connection with that claim. In that same suit, Plaintiff also filed a $105,000 tort claim against the president of Books, Inc., a citizen of New York, alleging that he had intentionally inflicted emotional distress upon her by sending emails to dozens of other publishers denouncing her as a horrible writer and a fraud and explaining that those were the reasons for his company’s refusal to publish her book. In this same action, Books, Inc. filed a claim against President seeking indemnity for any liability it might accrue in connection with Plaintiff’s copyright infringement claim against it. In response, President filed a breach of contract claim against Books, Inc. alleging that Books, Inc. had improperly withheld two weeks of salary from him totaling $65,000. Finally, President asserted a tort claim against his neighbor, claiming that her negligent maintenance of her home significantly depreciated the value of his property, for which he sought an injunction and $45,000 in damages.

Is President’s claim against his neighbor joinable?

A

No, because it is a tort claim alleging conduct that lowered the value of his property.

This is a third-party claim. Accordingly, joinability is determined by Rule 14(a), which only permits joinder of third-party claims for indemnity or contribution. Since this claim is not an indemnity or contribution claim, (but rather a claim alleging conduct that lowered the value of his property), it is not joinable.

80
Q

Plaintiff, a lifelong resident of Atlanta, Georgia, brought suit in federal district court in Atlanta against a publisher, Books, Inc., alleging copyright infringement under the federal copyright statute, for which she sought $100,000 in damages. Books, Inc. is incorporated under the laws of Delaware and has its principal place of business in New York City. Plaintiff also filed a claim against Books, Inc. alleging that it had breached its agreement with her to publish her book upon receipt of a manuscript. She sought $60,000 in damages in connection with that claim. In that same suit, Plaintiff also filed a $105,000 tort claim against the president of Books, Inc., a citizen of New York, alleging that he had intentionally inflicted emotional distress upon her by sending emails to dozens of other publishers denouncing her as a horrible writer and a fraud and explaining that those were the reasons for his company’s refusal to publish her book. In this same action, Books, Inc. filed a claim against President seeking indemnity for any liability it might accrue in connection with Plaintiff’s copyright infringement claim against it. In response, President filed a breach of contract claim against Books, Inc. alleging that Books, Inc. had improperly withheld two weeks of salary from him totaling $65,000. Finally, President asserted a tort claim against his neighbor, claiming that her negligent maintenance of her home significantly depreciated the value of his property, for which he sought an injunction and $45,000 in damages.

Is President a properly joined defendant in this case?

A

Yes, because this claim is transactionally related to Plaintiff’s claim against Books, Inc. and the two claims share a common question of fact.

Since Plaintiff filed claims against two defendants, this raises the issue of joinder of parties-defendant. Under Rule 20(a) multiple defendants can be joined if the claim against them arises out of the same transaction and there is at least one question of either law or fact common to the claim against all joined defendants. Both of these requirements are met here as both claims arise out of the decision not to publish Plaintiff’s book and involve the common question of why the book was not published, among others.

81
Q

Plaintiff is an insurance company incorporated in Delaware. Defendant, its insured, owns an interstate bus company that is incorporated and headquartered in California. A Driver who works for Defendant crashed into a car on an interstate highway. More than a dozen individuals, both drivers and passengers, were seriously injured and three people eventually die from their injuries. Defendant was insured for claims up to $1 million for accidents that occur in the course of Driver’s employment. Driver had no personal insurance at the time of the accident. Ten of the injured parties sued Defendant and Driver, and their claims, consolidated in a federal district court in California, seek more than $10 million. Plaintiff filed a statutory interpleader action in federal court in Delaware.

Under what circumstances must the court dismiss the interpleader action?

A

The district court must dismiss if Plaintiff fails to deposit the stake into court or refuses to pay a bond.

Federal law recognizes two different kinds of interpleader actions: “Rule 22” interpleader and “statutory” interpleader (28 U.S.C. § 1335). Although the procedures serve the same purpose—to enable a party to avoid the risk of multiple and conflicting liability “when several claimants assert rights to a single stake,” Airborne Freight Corp. v. U.S., 195 F.3d 238, 240 (5th Cir. 1999)—statutory and rule interpleader differ in their procedural requirements. Statutory interpleader requires that the plaintiff deposit the stake “into the registry of the court, there to abide the judgment of the court, or has given bond payable to the clerk of the court in such amount and with surety as the court or judge may deem proper.” 28 U.S.C. § 1335(a)(1)(B); see U.S. Fire Ins. Co. v. Asbestospray, Inc., 182 F.3d 201, 210 (3d Cir. 1999) (“A proper deposit or bond is a jurisdictional prerequisite… The stakeholder …must deposit the largest amount for which it may be liable in view of the subject matter of the controversy.”)

82
Q

A citizen of Indiana brings an action against her employer, an Indiana corporation, in federal district court. Claim I alleges that she was discriminated against on the basis of her sex in violation of the federal Civil Rights Act of 1964. The second claim consists of a tort claim for negligent infliction of emotional distress arising out of the same series of acts of sexual harassment that formed the basis of her federal sex discrimination claim. The plaintiff seeks $100,000 in compensatory damages and $1 million in punitive damages under each of these two claims. The availability of punitive damages in negligent infliction of emotional distress cases has not previously been addressed under the governing Indiana state law.

Should the court exercise subject matter jurisdiction over the entire lawsuit?

A

No, not over the state law claim because it raises a novel issue of state law.

The federal claim falls within the court’s §1331 jurisdiction. The non-diverse state claim is subject to supplemental jurisdiction under §1367 if the two claims form part of the same case, i.e., arise out of a common nucleus of operative fact AND if none of the factors listed in §1367(c) justify declining to exercise such supplemental jurisdiction. Here, the two claims do form part of the same case, but since the damages issue under state law is a question of first impression, this falls within the §1367(c)(1) factor justifying a decision to decline to exercise supplemental jurisdiction over the state claim.

83
Q

*A citizen of Indiana brings an action against her employer, an Indiana corporation, in federal district court. Claim I alleges that she was discriminated against on the basis of her sex in violation of the federal Civil Rights Act of 1964. The second claim consists of a tort claim for intentional infliction of emotional distress arising out of the same series of acts of sexual harassment that formed the basis of her federal sex discrimination claim. The plaintiff seeks $250,000 in damages in connection with each claim.

Does the court have subject matter jurisdiction over the entire lawsuit?

A

A federal court must have subject matter jurisdiction over every claim in the lawsuit. The federal claim clearly falls within the court’s §1331 jurisdiction. The second claim, however, is a non-diverse state law claim since both parties are citizens of Indiana. Thus, it does not fall within the court’s original jurisdiction under either §1331 or §1332. This, then, raises the question of whether the claim falls within the court’s supplemental jurisdiction as provided by 28 U.S.C. §1367.

This is a “pendent claim jurisdiction” first articulated by the Supreme Court in UMW v. Gibbs. It consists of a federal question claim and a non-diverse state claim brought by one plaintiff against one defendant. Congress codified this doctrine in §1367(a), which gives federal courts the discretion to exercise supplemental jurisdiction under these circumstances as long as the two claims form part of the same “case” under Article III of the Constitution, i.e., they arise out of a common nucleus of operative facts. If they do not, then the state claim must be dismissed and the federal court retains jurisdiction over the federal claim only. If they do form part of the same “case,” then the court has the discretion to exercise jurisdiction over the (non-diverse state law) claim that did not fall within its original jurisdiction. And §1367(c)(1)-(4) lists the factors that the court should consider in deciding whether or not to decline to exercise this supplemental jurisdiction over the non-diverse state law claim.

In this question, both claims arise out of the same series of alleged acts of harassment and so the common nucleus test is satisfied. And none of the factors listed in §1367(c) such as predominance of state issues or novelty of state law issues or dismissal of the federal claim is present to justify dismissal of the state law claim. Therefore, the fact that there is no independent subject matter jurisdiction over the non-diverse state law claim is not fatal`

84
Q

*A purchaser of a television brought an action against the manufacturer in federal district court in Massachusetts containing a claim that the manufacturer participated in a conspiracy to engage in unlawful price-fixing in violation of the federal antitrust laws. The plaintiff is a citizen of Massachusetts and the defendant is a citizen of Delaware and Massachusetts. In his complaint, the plaintiff also asserted a breach of contract claim, alleging that the television was defective and that the manufacturer refused to honor the warranty contained in the purchase agreement. In connection with this second claim, the plaintiff seeks $100,000 in damages.

Can the court exercise subject matter jurisdiction over the entire lawsuit?

A

No, because the claims do not arise out of a common nucleus of operative fact.

The federal claim falls within the court’s §1331 jurisdiction. The non-diverse state claim (both parties are citizens of Massachusetts) is subject to supplemental jurisdiction under §1367 only if the two claims form part of the same case, meaning they arise out of a common nucleus of operative facts and if none of the factors listed in §1367(c)(1)-(4) justify declining to exercise such supplemental jurisdiction. Here, the facts giving rise to the price-fixing conspiracy are different from and unrelated to the facts giving rise to the breach of contract claim; the two claims do not form part of the same case. Therefore, there can be no supplemental jurisdiction over the non-diverse state law claim.

85
Q

Plaintiff is injured in a crash between his car and another car and a truck on a narrow country road in Concord, New Hampshire. Plaintiff is a citizen of New Hampshire. Driver, the driver of the other car, is a citizen of Maine, and the owner of the truck, Trucks, Inc., is incorporated under the laws of Delaware and has its principal place of business in Connecticut. Plaintiff filed an action in the federal court for the District of New Hampshire seeking damages of $250,000 against each of the two named defendants, Driver and Trucks, Inc. In the same suit, Driver filed a claim against Trucks, Inc. seeking $75,000 in damages for the damage to her car. Trucks, Inc. filed a tort claim against Plaintiff seeking $70,000 in damages for the damages to its truck. Trucks, Inc. also filed a claim against Trucker, a citizen of Connecticut who was driving its truck at the time of the accident, seeking indemnity from any loss it might suffer in connection with Plaintiff’s claim against it.

Can the court exercise subject matter jurisdiction over Trucks, Inc.’s claim against Plaintiff?

A

Yes, because this claim arose out of the car crash.

This is a state law counterclaim between diverse parties but as to which the amount in controversy requirement has not been met. So there is no original jurisdiction over this claim. But since original jurisdiction over the civil action is founded solely on §1332, whether or not the court can exercise supplemental jurisdiction over Trucks, Inc.’s counterclaim involves the application of §1367(b). This section prohibits the exercise of supplemental jurisdiction in diversity-based cases, but only over claims by “plaintiffs.” Although Trucks, Inc. is a counter-plaintiff, this does not meet the §1367(b) requirement of being a plaintiff. Trucks, Inc. is a defendant and counter-plaintiff and so the limitation of §1367(b) is inapplicable here. So we must go to §1367(a) and ask if this claim forms part of the same “case” as claims over which the court has original jurisdiction. And it does, since this claim arises out of the accident that gave rise to the plaintiff’s claims. So the common nucleus of operative fact standard of §1367(a) is met.

86
Q

Plaintiff is injured in a crash between his car and another car and a truck on a narrow country road in Concord, New Hampshire. Plaintiff is a citizen of New Hampshire. Driver, the driver of the other car, is a citizen of Maine, and the owner of the truck, Trucks, Inc., is incorporated under the laws of Delaware and has its principal place of business in Connecticut. Plaintiff filed an action in the federal court for the District of New Hampshire seeking damages of $250,000 against each of the two named defendants, Driver and Trucks, Inc. In the same suit, Driver filed a claim against Trucks, Inc. seeking $75,000 in damages for the damage to her car. Trucks, Inc. filed a tort claim against Plaintiff seeking $70,000 in damages for the damages to its truck. Trucks, Inc. also filed a claim against Trucker, a citizen of Connecticut who was driving its truck at the time of the accident, seeking indemnity from any loss it might suffer in connection with Plaintiff’s claim against it.

Can the court exercise subject matter jurisdiction over Driver’s claim against Trucks, Inc.?

A

Yes, because this claim arose out of the car crash.

Driver and Trucks, Inc. are diverse, but since the amount in controversy is only $75,000, it does not exceed $75,000 and therefore does not meet the amount in controversy requirement of §1332. Since this is a state law claim, the court does not have original jurisdiction over this cross-claim. Since original jurisdiction over the civil action is founded solely on §1332, whether or not the court can exercise supplemental jurisdiction over Driver’s claim involves the application of §1367(b). This section prohibits the exercise of supplemental jurisdiction in diversity-based cases, but only over claims by “plaintiffs.” Although Driver is a cross-plaintiff, this does not meet the §1367(b) requirement of being a plaintiff. She is a defendant and cross-plaintiff and so the limitation of §1367(b) is inapplicable here. So we must go to §1367(a) and ask if this claim forms part of the same “case” as claims over which the court has original jurisdiction. And it does, since this claim arises out of the accident that gave rise to the plaintiff’s claims. So the common nucleus of operative fact standard of §1367(a) is met.

87
Q

Plaintiff is injured in a crash between his car and another car and a truck on a narrow country road in Concord, New Hampshire. Plaintiff is a citizen of New Hampshire. The driver of the other car is a citizen of Maine, and the owner of the truck, Trucks, Inc., is incorporated under the laws of Delaware and has its principal place of business in Connecticut. Plaintiff filed an action in the federal court for the District of New Hampshire seeking damages of $250,000 against each of the two named defendants, Driver and Trucks, Inc. In the same suit, Driver filed a claim against Trucks, Inc. seeking $75,000 in damages for the damage to her car. Trucks, Inc. filed a tort claim against Plaintiff seeking $70,000 in damages for the damages to its truck. Trucks, Inc. also filed a claim against Trucker, a citizen of Connecticut who was driving its truck at the time of the accident, seeking indemnity from any loss it might suffer in connection with Plaintiff’s claim against it.

Can the court exercise subject matter jurisdiction over Trucks, Inc.’s claim against Trucker?

A

Yes, because this claim arose out of the car crash.

This is a state law claim between non-diverse parties so there is no original jurisdiction over this claim. It is a third-party claim in a case where original jurisdiction was founded solely on diversity. Thus, we need to look to §1367(b), but the ban on supplemental jurisdiction in §1367(b) only applies to claims by plaintiffs and this is a third-party claim by a defendant and so the limitation of §1367(b) is inapplicable. We then turn to §1367(a), which would allow supplemental jurisdiction if this claim forms part of the same case as the claims within the court’s original jurisdiction. Since this third-party claim is for indemnity for any loss suffered by Trucks, Inc. in connection with the plaintiff’s claim against it, the common nucleus of operative fact standard of §1367(a) has been met and therefore the court can exercise supplemental jurisdiction.

88
Q

Plaintiff, a citizen from State N, brought a patent infringement claim against Avant Co., a Delaware corporation, in federal court in State N. Plaintiff alleged that Avant Co. had infringed her patent in violation of the federal patent statute and sought damages in the amount of $50,000. Plaintiff also asserted a $250,000 intentional infliction of emotional distress claim against the president of Avant, James Avant, a citizen of State N. In this claim, Plaintiff alleges that James Avant had subjected her to repeated series of acts of sexual harassment and that this caused Plaintiff to suffer severe emotional distress.

Defendant James Avant filed a motion to dismiss challenging the court’s exercise of jurisdiction over the supplemental claim. How should the court rule?

A

The court should grant the motion, because the two claims do not form part of the same case.

Under §1367(a) if the plaintiff asserts a federal question claim against one defendant, the court can exercise supplemental jurisdiction over a non-diverse state law claim against a separate defendant if the two claims form part of the same case, i.e., arise out of a common nucleus of operative fact, and if none of the factors listed in §1367(c) justify declining to exercise such supplemental jurisdiction. Here, the patent claim against the corporation is a federal question claim. The tort claim against the president, however, does not form part of the same case as the patent claim since it is based on acts of sexual harassment and the former claim arose out of the company’s acts of infringement. Thus, supplemental jurisdiction under §1367(a) is not available,

89
Q

Plaintiff, a citizen of Missouri, is a widow whose husband was killed in an accident while working for Defendant, a construction company. Defendant is incorporated and has its principal place of business in Idaho. Plaintiff brought a diversity action against Defendant in federal district court in Missouri alleging negligence and seeking $100,000 in damages. Defendant filed a third-party claim against Distributor, seeking $100,000 in damages and alleging that if Defendant is liable to Plaintiff, Distributor is liable to Defendant for having leased the equipment that allegedly caused the accident. Distributor is incorporated in Missouri. Plaintiff seeks to amend her complaint to assert a tort claim against Distributor and to seek $100,000 in damages from that party.

Defendant and Distributor each move to dismiss the claims against them based on subject-matter jurisdiction objections. Defendant moves to dismiss Plaintiff’s negligence claim; Distributor moves to dismiss Defendant’s third-party claim; and Distributor also moves to dismiss Plaintiff’s tort claim. How should the court resolve the motions?

A

Grant Distributor’s motion against Plaintiff.

Plaintiff’s claim against Distributor is not within the court’s diversity jurisdiction; supplemental jurisdiction cannot be exercised over it; and the facts do not suggest that any other jurisdictional basis is available. When faced with a multi-party and multi-claim action, it helps to diagram the action: Plaintiff (Missouri) v. Defendant (Idaho) v. Distributor (Missouri) Here, Plaintiff is seeking to amend her complaint to sue Distributor, who has been joined in the action as a third-party defendant, under FRCP 14.

However, in order to bring a claim against a third-party defendant, the court must be able to exercise subject-matter jurisdiction. One form of subject-matter jurisdiction is diversity jurisdiction under 28 U.S.C. § 1332(a). The federal diversity statute permits the exercise of diversity jurisdiction if the plaintiff and the defendant are “citizens of different States” and “the matter in controversy exceeds the sum or value of $75,000 exclusive of interests and costs.” 28 U.S.C. § 1332(a)(1). Plaintiff (Missouri) and Distributor (Missouri) are citizens of the same state, and so diversity jurisdiction is not present. The second form of subject-matter jurisdiction is supplemental jurisdiction, though this too is unavailable in this situation.

Here, Plaintiff and Defendant are citizens of different states and the amount-in-controversy exceeds $75,000, and so diversity jurisdiction is present. When the anchor claim is grounded in diversity jurisdiction, federal law withholds supplemental jurisdiction over any claim “by plaintiffs against persons made parties under Rule 14, 19, 20, or 24 of the Federal Rules of Civil Procedure” 28 U.S.C. § 1367(b). Defendant’s third-party claim against Distributor requires that Distributor be joined under FRCP Rule 14 which governs third-party practice in federal court. Thus, Distributor is a person made a party under FRCP 14, and Plaintiff is prohibited from joining a claim under § 1367(b); supplemental jurisdiction may not be exercised on Plaintiff’s claim against Distributor.

The statute codifies the practice that existed before enactment of 28 U.S.C. § 1367, see Owen Equip. & Erection Co. v. Kroger, 437 U.S. 365 (1978), which did not permit ancillary jurisdiction when its exercise would violate the rule of complete diversity as articulated in Strawbridge v. Curtiss, 7 U.S. 267 (1806).

90
Q

Plaintiff, a State A resident, worked for a Delaware corporation for over 25 years. After working tirelessly for six months in anticipation of a promotion, Plaintiff was informed that the company had hired a new employee for the coveted position. Believing that she had been passed over based on her age and gender, Plaintiff filed suit against the corporation under federal and state anti-discrimination laws. Plaintiff also filed a claim against her district manager, a State A resident, alleging breach of employment contract, seeking $50,000 in damages. Plaintiff filed her action in federal court in State A. The district manager moved to dismiss for lack of subject matter jurisdiction. How should the court rule on this motion?

A

The court should deny the motion, and in its discretion exercise supplemental jurisdiction over the contract claim against the district manager.

Supplemental jurisdiction provides federal court jurisdiction over claims which lack an independent basis for federal subject matter jurisdiction but that arise from the same set of operative facts as the principal claim validly asserted in federal court. Under § 1367(a), “in any civil action of which the district courts have original jurisdiction, the district courts shall have supplemental jurisdiction over all other claims that are so related to claims in the action within such original jurisdiction that they form part of the same case or controversy under Article III of the United States Constitution.” The exercise of such power is subject to exceptions set out in § 1367(b) when the original claim is based on diversity jurisdiction. Here, the court’s original jurisdiction is based on a claim that arises under federal law; the claim against the district manager is related to that claim because it involves the conditions of Plaintiff’s employment and loss of promotion at work, and thus falls under the federal court’s supplemental jurisdiction.

91
Q

Plaintiffs, both citizens of State N, filed an action in federal court in Delaware against Corp. Inc., a Delaware corporation with its principal place of business in State N, alleging that the prospectus issued by Corp. Inc. in connection with the sale of its stock contained misrepresentations in violation of both federal securities law and Delaware corporate law. In connection with that claim, Plaintiffs each sought $250,000 in damages.

In the same lawsuit, Plaintiffs also asserted a $300,000 claim against Stock Co., a stock brokerage firm, incorporated in State N, alleging that Stock Co. had distributed these prospectuses in full knowledge of the existence of these material misrepresentations in violation of Delaware law. Stock Co. in turn filed a $500,000 breach of contract claim against Corp. Inc. alleging that Corp. Inc. failed to pay its commission earned by selling shares of Corp. Inc. Corp. Inc. filed a motion to dismiss the breach of contract claim based on lack of subject matter jurisdiction. Stock Co. opposed the motion arguing that the federal court has jurisdiction over the breach of contract claim.

How should the court rule on the motion?

A

The court should grant the motion, because the claim is unrelated to the plaintiffs’ claims against Corp. Inc.

The claim is a cross-claim brought by one defendant against a co-defendant. It is a state law claim brought by one N citizen against another N citizen. Therefore there is no original jurisdiction over this claim under either federal question (§1331) or diversity (§1332). A cross-claim, such as the one at issue here, may fall under the court’s supplemental jurisdiction (§1367), provided it forms part of the same case as the claim that falls within the court’s original jurisdiction, (here, Plaintiffs’ federal securities claims against Corp. Inc.). This breach of contract claim from Stock Co. against Corp. Inc. is not part of the same case. Thus, it does not fall under the court’s supplemental jurisdiction.

92
Q

A plaintiff sued a manufacturer and retailer on a defective-design claim in federal district court. The retailer wanted to add a claim against the manufacturer for violations of the Sherman Antitrust Act. The retailer told its lawyer that it believed the manufacturer had conspired with others in the market to illegally fix prices for the product that had injured the plaintiff.

Is the federal district court likely to permit joinder of the retailer’s claim?

A

No, because the retailer’s antitrust claim does not arise from the same transaction or occurrence as the plaintiff’s defective-design claim.

When one party asserts a claim against another coparty (i.e., another party on the same side of the “v.”), that claim is called a crossclaim. A crossclaim must arise out of the same transaction or occurrence that is the subject of either the original action or of a counterclaim. Fed. R. Civ. P. 13(g).

Here, the manufacturer and the retailer are both defendants and are therefore coparties. The retailer’s claim against the manufacturer is a crossclaim. The district court is unlikely to permit joinder because the retailer’s claim does not arise out of the same transaction or occurrence as the plaintiff’s defective-design claim. Besides involving the same product, there is no logical relationship between the plaintiff’s claim that the product was defectively designed and the retailer’s claim that the manufacturer conspired to fix the product’s price. The two claims would likely involve substantially different evidence.

93
Q

Two employees filed a lawsuit against their employer, alleging violations of federal and state overtime regulations. A third employee sought to join the litigation to bring a single claim against the same employer for recovery of medical expenses incurred as a result of injuries she sustained while operating a forklift during ordinary working hours.

What is most likely the court’s ruling on the third employee’s request to join the lawsuit?

A

The court will sever the claim, because the claim does not share common factual or legal issues with the existing lawsuit, and the claims do not arise out of the same transaction or occurrence.

Federal Rule of Civil Procedure 20 governs the possible permissive joinder of parties to an existing case. Specifically, Rule 20(a)(1) provides that a new party may join in an existing action as a plaintiff if: (1) the new party’s claim arises out of the same transaction or occurrence as the existing case, and (2) there is a commonality in a factual or legal issue that will apply to all the plaintiffs (the existing plaintiffs and the newly joined plaintiff). If a claim fails to comply with the requirements for joinder under Rule 20 of the Federal Rules of Civil Procedure, then the court may sever the claim so that it is brought in a separate lawsuit. Fed. R. Civ. P. 21.

94
Q

A plaintiff sued a retailer in federal district court. The complaint alleged that a product the plaintiff had purchased from the retailer was defectively designed and injured the plaintiff. The retailer moved to dismiss the plaintiff’s complaint for failure to state a claim under Rule 12(b)(6). After briefing and oral argument one month later, the district court denied the motion. The retailer then served and filed a third-party complaint against the product’s manufacturer. The governing substantive law on products liability gave the retailer a right of indemnification against the manufacturer.

Is the retailer’s third-party complaint properly filed?

A

Yes, because the substantive law provides the retailer a right of indemnification against the manufacturer.

Rule 14 of the Federal Rules of Civil Procedure allows a defending party to join a third party who is or may be liable to the defending party for some or all of the claim against the defending party. Fed. R. Civ. P. 14(a). This procedure is called impleader, or third-party practice.

Here, the substantive law provides the retailer with a right of indemnification against the manufacturer. The manufacturer’s liability to the retailer is derivative: the manufacturer is only liable to the retailer if the retailer is liable to the plaintiff. This is precisely the type of situation for which third-party practice was designed. The retailer’s third-party complaint is therefore properly filed.

95
Q

After being injured in a three-vehicle accident, a plaintiff sued a truck driver for negligence in federal district court. The accident occurred when a minivan driver failed to stop in time and rear-ended the truck driver. The impact of the rear-end collision, in turn, caused the truck driver to rear-end the plaintiff. The plaintiff chose not to sue the minivan driver. The truck driver felt that the minivan driver held most of the fault for the collision. The truck driver did not want to be left paying for all of the plaintiff’s alleged injuries, so he asked his lawyer if there was a way to bring the minivan driver into the lawsuit.

Where should the lawyer look first to determine whether the truck driver may add the minivan driver to the lawsuit?

A

The lawyer should check whether, under substantive law, the truck driver has a right of indemnification or contribution from the minivan driver.

96
Q

A plaintiff sued a defendant in federal district court for breach of contract. The defendant asserted a counterclaim against the plaintiff, also for breach of contract, alleging that the plaintiff did not deliver the goods being purchased under the contract in a timely fashion. The plaintiff wanted to add its supplier of the goods to the lawsuit. The plaintiff told its lawyer that the supplier’s delivery of the goods was what had dictated the timing of the plaintiff’s delivery to the defendant: in other words, that any lateness on the plaintiff’s part was the supplier’s fault. The plaintiff’s contract with the supplier gave the plaintiff a right of indemnification against the supplier for any lawsuit subsequently involving the goods.

May the plaintiff use impleader to add its indemnification claim against the supplier to the lawsuit?

A

Yes, because the plaintiff is a counterclaim-defendant.

Impleader may be used by any defending party to join a nonparty who is or may be liable to the defending party for all or part of the claim against it. Fed. R. Civ. P. 14(a)(1). A party need not be an original defendant in order to qualify as a defending party under Rule 14. When a claim is asserted against a plaintiff, a plaintiff may use impleader to bring in a third party if Rule 14 would permit a defendant to do so. Fed. R. Civ. P. 14(b).

Here, the defendant has asserted a counterclaim for breach of contract against the plaintiff. Because a claim has been asserted against the plaintiff, the plaintiff may use impleader to assert a third-party complaint against its supplier

97
Q

A plaintiff filed a lawsuit in federal district court against a contractor, alleging state-law claims based on poor workmanship in her home renovation. She sought $100,000 in damages. The defendant contractor then filed a third-party claim pursuant to Federal Rule of Civil Procedure 14 against his subcontractor (a corporation), alleging that the subcontractor was responsible for any damages that the plaintiff might be awarded in the case, because the subcontractor’s workers had performed the work that the plaintiff alleged was substandard. The plaintiff was a citizen of Louisiana, the defendant contractor was a citizen of Mississippi, and the third-party defendant subcontractor was a citizen of Louisiana and Mississippi.

Does the court have supplemental jurisdiction over the third-party claim between the contractor and subcontractor?

A

Yes, because 28 U.S.C. § 1367(a) is satisfied and § 1367(b) does not apply to a third-party claim brought by a defendant.

Here, the only basis for subject-matter jurisdiction is diversity jurisdiction. Diversity jurisdiction exists over the plaintiff’s claim against the defendant contractor, because the parties are diverse and the amount in controversy exceeds $75,000. The third-party claim arises from the same renovation and alleged poor workmanship as the plaintiff’s claim against the defendant contractor. Thus, the defendant contractor’s third-party claim against the subcontractor falls within the general rule set forth in 28 U.S.C. § 1367(a), which permits the court to exercise supplemental jurisdiction over a claim that forms “part of the same case or controversy” when it shares a “common nucleus of operative fact” with the claim that has an independent basis for federal subject-matter jurisdiction.

Section 1367(b) does not apply because it is not the plaintiff (or a person proposed to be joined, or seeking to intervene, as a plaintiff) who is asserting the claim, but rather the defendant. Remember, defendants are generally considered “defendants” for many jurisdictional statutes (including 28 U.S.C. § 1367(b)), even if they become “plaintiff-like” by actively asserting claims against plaintiffs or third parties.

98
Q

A homeowner sued a housepainter for breach of contract in federal district court. The housepainter had painted the home’s interior six months before the lawsuit, and the paint had begun to peel all over the home. The housepainter’s answer alleged that the cause of the peeling was a defect in the paint, which he had provided but not designed. Together with the answer, the housepainter filed a motion to dismiss under Rule 12(b)(7) for failure to join a required party, namely, the paint manufacturer.

Is the district court likely to grant the housepainter’s motion?

A

No, because the paint manufacturer is not a necessary party.

In deciding a motion to dismiss for failure to join a required party, a court must determine whether the omitted party is necessary, and whether it is feasible to join the omitted party. If the omitted party is necessary but it is not feasible to join them, then the court must decide whether to proceed with the lawsuit in the party’s absence or dismiss it. Fed. R. Civ. P. 19. A party is necessary if the court cannot provide complete relief in the party’s absence, or if the party has an interest in the case and proceeding without the party may either impair the party’s ability to protect that interest, or leave an existing party subject to multiple or inconsistent liabilities. Fed. R. Civ. P. 19(a)(1)(A), (B).

The paint manufacturer is not a necessary party. The homeowner may obtain complete relief from the housepainter. The paint manufacturer has no interest in the lawsuit that requires protecting, nor does proceeding in the absence of the manufacturer subject the housepainter to multiple or inconsistent liabilities. Therefore, the district court is not likely to grant the housepainter’s motion. Rather, the housepainter would be better served here by filing a third-party complaint against the paint manufacturer, provided that the underlying substantive law gives the housepainter a right of indemnification or contribution. See Fed. R. Civ. P. 14(a)(1).

99
Q

A vacation homeowner, a citizen of State A, sued the local county government in State B for continued use of a right-of-way easement. The easement, which the vacation homeowner had used each summer, stretched from the vacation home across county-owned land to the beachfront. In preparation for a sale of a portion of the land, the county had fenced off a portion of the land over which the easement stretched. The fence barred the vacation homeowner’s easement path to the beach. Shortly before the homeowner filed suit, the county had sold a portion of the fenced-in land to a private developer, a citizen of State A. The homeowner had not used the existing easement since the fence was constructed. However, the homeowner had other paths by which to reach the beachfront without using the developer’s land or needing to cross the fenced-in area. The private developer was not named in the homeowner’s lawsuit. The county moved to dismiss the vacation homeowner’s lawsuit under Rule 12(b)(7) for failure to join a required party.

Which of the following facts best supports the district court’s decision to proceed without the private developer?

A

The homeowner can reach the beachfront from the homeowner’s lot without traveling over the portion of land the private developer purchased.

If it is not feasible to join a necessary party to a lawsuit, a federal district court has two options: (1) proceed with the lawsuit in the party’s absence or (2) dismiss the lawsuit. Fed. R. Civ. P. 19. In general, courts consider four nonexclusive factors to decide whether to proceed or dismiss, including whether: (1) a judgment without the required person would prejudice that person or other parties, (2) a judgment without the required person could be crafted to avoid any prejudice, (3) a judgment without the required person would be adequate to resolve the dispute, and (4) the plaintiff has an adequate remedy if the case were dismissed. Fed. R. Civ. P. 19(b)(1)-(4).

Here, the fact that there are routes between the vacation home and the beachfront that do not cross the private developer’s land indicates that the court may craft an adequate remedy for the homeowner without prejudicing the private developer. See Fed. R. Civ. P. 19(b)(2). This fact best supports a court’s decision to proceed with the lawsuit in the private developer’s absence.

100
Q

An engineer resigned from a technology company. She formed her own company and continued using the same technical designs that she had used during her previous employment. The technology company sued the engineer in federal court, seeking damages for using its patented designs in breach of her employment contract. The technology company claimed it held an exclusive license to use the designs, granted by the patent holder. The engineer filed counterclaims against the technology company, seeking a declaration that the patent was invalid and that the designs were in the public domain. The technology company filed a motion to dismiss the engineer’s counterclaim for failure to join an indispensable party. In ruling on the motion, the court determined that the patent holder was a necessary party, but that joinder was not feasible.

Which of the following actions should the court take next?

A

The court should determine whether the patent holder is an indispensable party.

After determining that the patent holder is a necessary party whose joinder is not feasible, the court must determine whether the patent holder is an indispensable party. Fed. R. Civ. P. 19(a)(2) and 19(b). That inquiry requires the court to determine whether, in equity and good conscience, the action should proceed among the existing parties or should be dismissed. Fed. R. Civ. P. 19(b). Only after making that determination may the court rule on a motion to dismiss for failure to join an indispensable party.

101
Q

A plaintiff sued three defendants for breach of warranty, breach of contract, and fraud in federal district court. Following extensive discovery, the district court denied all pending motions for summary judgment, concluding that the plaintiff’s claims required a jury trial. Two weeks before trial was scheduled to begin, one of the defendants moved for leave to implead a third-party defendant. The defendant argued that underlying substantive law gave the defendant a right of indemnification against the third-party defendant for the breach-of-warranty claim.

What best explains the district court’s decision to deny the defendant’s motion for leave to implead the third-party defendant?

A

The defendant’s delay in bringing the third-party complaint was excessive.

In deciding whether to permit or deny impleader, a court will generally balance efficiency against prejudice. Specifically, the court will balance the extent to which impleader will save time, effort, and duplicative litigation on the one hand, against any prejudice or harm that might result to the parties on the other hand. The prejudice or harm might take the form of excessive introduction of issues unrelated to the main claim, unexplained or excessive delay in bringing the third-party complaint, or undue complication of the lawsuit.

the district court’s decision to deny the defendant’s motion for leave to implead a third-party defendant is best explained by the defendant’s excessive delay in bringing the third-party complaint. Trial on the plaintiff’s claims is set for two weeks. Joining a third-party defendant at this late stage in the litigation would prejudice not only the third-party defendant but also the other parties to the action.

102
Q

A patient brought a diversity action in federal court against a medical device manufacturer for injuries sustained from a device that malfunctioned. The patient claimed that the malfunction was a result of the manufacturer’s negligent design of the device. The manufacturer claimed that the surgeon and hospital were joint tortfeasors and were liable for some or all of the patient’s claimed damages. Joinder of the surgeon and hospital would defeat diversity jurisdiction.

Are the surgeon and hospital necessary parties who must be joined?

A

No, because subsequent proceedings can resolve claims of indemnification or contribution.

To determine whether a party is necessary under Rule 19 of the Federal Rules of Civil Procedure (FRCP), courts determine whether the court can accord complete relief among the existing parties to the litigation without joinder of the missing party, or whether proceeding without the missing party would subject the existing parties to multiple or inconsistent obligations because of the missing party’s interest in the subject matter of the litigation. Joint tortfeasors are not considered necessary parties.

Here, failure to join the surgeon and hospital would not negatively subject either the manufacturer or the missing parties to multiple obligations. Any claims for indemnification or contribution may be decided in a subsequent proceeding after resolution of the manufacturer’s liability.

103
Q

A group of property owners in a vacation development brought a diversity action against the property management company in federal court, seeking an injunction against enforcement of parking restrictions recently enacted by the homeowners’ association. The property owners claimed that the restrictions were improperly enacted and violated the association’s bylaws. The management company filed a motion to dismiss for failure to join a necessary party under Federal Rule of Civil Procedure 19. The management company argued that it would violate the terms of its agreement with the homeowners’ association if it failed to enforce the enacted restrictions, and that the association had an interest in the interpretation of its bylaws and in the validity of the restrictions it had enacted. Venue is proper, the court has personal jurisdiction over the homeowners’ association, and the association can be properly served. Joining the association would not affect the court’s diversity jurisdiction.

How is the court likely to rule on the motion to dismiss?

A

The court will order joinder of the homeowners’ association, because the association is a necessary party and joinder is feasible.

The court must determine whether the homeowners’ association is a necessary party under Federal Rule of Civil Procedure 19(a). Rule 19(a) requires joinder, if feasible, where (1) the missing party’s absence prevents the court from giving the existing parties complete relief; or where (2) the missing party claims an interest in the litigation, and disposing of the action in the party’s absence may impair its ability to protect that interest or create a substantial risk of inconsistent obligations. Here, the homeowners’ association has an interest in both the interpretation of its bylaws and the validity of its restrictions. If the association is not joined, the court’s judgment could subject the management company to obligations that diverge from its contractual obligations to the association. Thus, the association is necessary and must be joined if feasible. The facts state that venue is proper, the association could be properly served, and the court would have personal jurisdiction over the association without defeating diversity jurisdiction after joinder. Therefore, joinder of the association is feasible. Because it is both necessary and feasible, the court must order joinder of the association.

104
Q

A plaintiff sued two competitors in federal district court for tortious interference with contract, a state-law claim. The complaint alleged that the competitors had poached the plaintiff’s customers by misrepresentation and fraud. The plaintiff was a citizen of State A, and both competitors were citizens of State B. The plaintiff alleged $500,000 in damages as a result of the competitors’ tortious interference. In discussing strategy, the larger competitor instructed its lawyer that it would not cooperate in any joint defense, and in fact wanted to sue the smaller competitor. The larger competitor explained that it believed the smaller competitor had engaged in false advertising that spread misinformation about the larger competitor’s products. The false advertising had not involved the plaintiff’s products or the customers that the plaintiff claimed to have lost as a result of both competitors’ conduct. The larger competitor estimated that its revenues had dropped almost $100,000 in the past year since the smaller competitor began the advertising campaign. The larger competitor’s lawyer was researching the possibility of a claim for unfair competition under state law.

Which of the following is most accurate about the larger competitor’s potential unfair-competition claim against the smaller competitor?

A

The district court would not have subject-matter jurisdiction over the claim and joinder would not be appropriate.
In evaluating whether a potential claim may be added to a particular lawsuit, one should consider whether the court has subject-matter jurisdiction over the claim and whether joinder of the claim is appropriate.

In order for a federal district court to exercise subject-matter jurisdiction over a claim arising under state law, the requirements for diversity jurisdiction or supplemental jurisdiction must be met. Diversity jurisdiction requires that the parties on opposite sides of the claim be completely diverse (i.e., be citizens of different states) and that the amount in controversy exceed $75,000 exclusive of interest and costs. 28 U.S.C. § 1332. If the requirements for diversity jurisdiction are not met, a district court may still exercise supplemental jurisdiction if the state-law claim shares a common nucleus of operative fact with another claim that has an independent jurisdictional basis, such that the new claim may be considered part of the same case or controversy as that within the district court’s subject-matter jurisdiction. 28 U.S.C. § 1367(a). If the court may exercise subject-matter jurisdiction over the new state-law claim, it must then ensure that the requirements for proper joinder of the claim are met.

A claim asserted by one party against a coparty (i.e., a party on the same side of the “v.”) is called a crossclaim. In order for joinder of a crossclaim to be proper, the crossclaim must arise out of the same transaction or occurrence that is the subject matter of the original claim or of a counterclaim. Fed. R. Civ. P. 13(g).

Here, the two competitors are both citizens of State B, so the larger competitor’s potential state-law claim for unfair competition against the smaller competitor does not meet the requirements for diversity jurisdiction. Moreover, the unfair-competition claim is not related to the plaintiff’s claim against the two competitors. The potential claim is based on different facts and has no logical relationship with the plaintiff’s underlying tortious-interference claim. Therefore, the district court may not exercise supplemental jurisdiction over the unfair-competition claim, either. For these reasons, the district court does not have subject-matter jurisdiction over the larger competitor’s state-law claim.

For similar reasons, joinder of the claim is not proper. The larger competitor’s crossclaim against the smaller competitor is based on different facts than the plaintiff’s claim for tortious interference. The crossclaim therefore does not arise out the same transaction or occurrence as the plaintiff’s tortious-interference claim. For this reason, joinder of the unfair-competition claim is not appropriate, either

105
Q

A plaintiff sued her former employer in federal district court for violating her civil rights under federal law. The complaint alleged that the former employer terminated the plaintiff’s employment due to her race. The former employer was meeting with its lawyer to discuss potential counterclaims it might assert against the plaintiff. Thus far, all of the potential counterclaims arose under state law, and the plaintiff and her former employer were citizens of the same state. The potential counterclaims did not require the addition of any new parties.

Would the court have jurisdiction over the potential counterclaims?

A

The district court will be able to exercise supplemental jurisdiction over any of the former employer’s compulsory counterclaims, even if they otherwise lack an independent jurisdictional basis, but any of the former employer’s permissive counterclaims must have an independent jurisdictional basis.

In evaluating whether a potential claim may be added to a particular lawsuit, one should consider whether the court has subject-matter jurisdiction over the claim and whether joinder of the claim is appropriate. In general, a federal district court may exercise subject-matter jurisdiction over a claim in one of three ways: (1) if the claim presents a federal question, (2) if the claim meets the requirements for diversity jurisdiction (because the parties are completely diverse and the amount in controversy exceeds $75,000), or (3) if the claim is within the court’s supplemental jurisdiction. See 28 U.S.C. §§ 1331, 1332, 1367(a). A district court may exercise supplemental jurisdiction if a state-law counterclaim shares a common nucleus of operative fact with another claim that has an independent jurisdictional basis, such that the counterclaim may be considered part of the same case or controversy as that within the district court’s subject-matter jurisdiction. See 28 U.S.C. § 1367(a).

A party may join as many counterclaims as it has against an opposing party. Fed. R. Civ. P. 18(a). Counterclaims may be either compulsory or permissive. A counterclaim is compulsory if it arises out of the same transaction or occurrence as the opposing party’s claim and the court has jurisdiction over any parties the claim requires to be added. Fed. R. Civ. P. 13(a)(1). A counterclaim arises out of the same transaction or occurrence as the opposing party’s claim if there is a logical relationship between the claims. In contrast, a permissive counterclaim is entirely unrelated to the opposing party’s claim. Fed. R. Civ. P. 13(b).

Here, because the former employer’s potential counterclaims are based upon state law, the district court would not have federal-question jurisdiction. The district court could not exercise diversity jurisdiction over the counterclaims either, because both the plaintiff and her former employer are citizens of the same state. Therefore, the only basis for subject-matter jurisdiction is supplemental jurisdiction.

A claim within the district court’s supplemental jurisdiction must share a common nucleus of operative fact with another claim that has an independent jurisdictional basis. Here, the only other claim in the case is the plaintiff’s federal civil-rights claim for employment discrimination. The plaintiff’s federal claim is also the opposing party’s claim with which any compulsory counterclaims must share a logical relationship in order for joinder to be appropriate. Therefore, by definition, any compulsory counterclaim properly joined against the plaintiff is within the court’s supplemental jurisdiction.

106
Q

An advocacy organization sued a city in federal district court. The organization’s complaint alleged that the city had violated the organization’s members’ rights under the First Amendment by denying the organization a permit for a planned demonstration. The organization’s headquarters were located in the city inside State A, where the organization was also incorporated. In conferring with its outside counsel, the head of the city’s legal department shared that the city had long suspected that the organization had fraudulently failed to pay city taxes. The city wanted to assert a counterclaim against the organization for fraud under state law, in an attempt to recover the back taxes owed. When asked for a back-of-the-envelope calculation, the city lawyer estimated that the organization had underpaid by as much as $1,000,000 cumulatively over the past five years.

How would a court determine if the city’s proposed counterclaim for fraud?

A

The district court would not have subject-matter jurisdiction over the claim but joinder would be appropriate.

In evaluating whether a potential claim may be added to a particular lawsuit, one should consider whether the court has subject-matter jurisdiction over the claim and whether joinder of the claim is appropriate. In general, a federal district court may exercise subject-matter jurisdiction over a claim in one of three ways: (1) if the claim presents a federal question, (2) if the claim meets the requirements for diversity jurisdiction (because the parties are completely diverse and the amount in controversy exceeds $75,000), or (3) if the claim is within the court’s supplemental jurisdiction. See 28 U.S.C. §§ 1331, 1332, 1367(a). A district court may exercise supplemental jurisdiction if a state-law claim shares a common nucleus of operative fact with another claim that has an independent jurisdictional basis, such that the claim may be considered part of the same case or controversy as that within the district court’s subject-matter jurisdiction. See 28 U.S.C. § 1367(a).

A party may join as many counterclaims as it has against an opposing party. Fed. R. Civ. P. 18(a). Counterclaims may be either compulsory or permissive. A counterclaim is compulsory if it arises out of the same transaction or occurrence as the opposing party’s claim and the court has jurisdiction over any parties the claim requires to be added. Fed. R. Civ. P. 13(a)(1). A counterclaim arises out of the same transaction or occurrence as the opposing party’s claim if there is a logical relationship between the claims. In contrast, a permissive counterclaim is entirely unrelated to the opposing party’s claim. Fed. R. Civ. P. 13(b).

Here, the city’s proposed counterclaim against the organization is for fraud under state law and does not present a federal question. The city and the organization are both citizens of State A, so the requirements for diversity jurisdiction are not met. Nor is supplemental jurisdiction appropriate here because the city’s fraud claim is entirely unrelated to the organization’s claim that the city violated its members’ rights under the First Amendment. The two claims are based on entirely different facts and are not part of the same case or controversy. The district court here could not exercise subject-matter jurisdiction over the city’s state-law fraud counterclaim.

107
Q

A plaintiff sued a retailer in federal district court. The complaint alleged that one of the retailer’s products was defectively designed and had injured the plaintiff. The plaintiff’s claim arose under state law, which also provided the retailer with a right of indemnification against the product’s manufacturer. The plaintiff was a citizen of State A and the retailer was a citizen of State B. The plaintiff alleged damages of $250,000 against the retailer only, and chose not to sue the manufacturer. The retailer filed a third-party complaint against the manufacturer, seeking indemnification for $50,000 of the plaintiff’s alleged damages. The manufacturer was a citizen of State B.

Does the district court have subject-matter jurisdiction over the retailer’s claim for indemnification against the manufacturer?

A

Yes, because the retailer’s claim against the manufacturer arises out of a common nucleus of operative fact as the plaintiff’s claim.

In evaluating whether a potential claim may be added to a particular lawsuit, one should consider whether the court has subject-matter jurisdiction over the claim and whether joinder of the claim is appropriate. In general, a federal district court may exercise subject-matter jurisdiction over a claim in one of three ways: (1) if the claim presents a federal question, (2) if the claim meets the requirements for diversity jurisdiction (because the parties are completely diverse and the amount in controversy exceeds $75,000), or (3) if the claim is within the court’s supplemental jurisdiction. See 28 U.S.C. §§ 1331, 1332, 1367(a). A district court may exercise supplemental jurisdiction if a state-law claim shares a common nucleus of operative fact with another claim that has an independent jurisdictional basis, such that the state-law claim may be considered part of the same case or controversy as that within the district court’s subject-matter jurisdiction. See 28 U.S.C. § 1367(a). Rule 14 permits third-party claims against a nonparty who may be liable to a defending party for part or all of the claims against it. Fed. R. Civ. P. 14(a)(1).

Here, the retailer’s third-party claim against the manufacturer arises under state law and does not present a federal question. Nor is diversity jurisdiction appropriate, because the retailer and the manufacturer are both citizens of State B. As such, the only basis for the district court’s subject-matter jurisdiction is supplemental jurisdiction. Here, the district court may exercise supplemental jurisdiction because the retailer’s claim for indemnification arises out of a common nucleus of operative fact as the plaintiff’s claim against the retailer. All of the claims are bound up in the plaintiff’s injury from the product the retailer sold and the manufacturer made. Both the retailer’s third-party complaint and the plaintiff’s original complaint would therefore be based on a common nucleus of operative fact, and supplemental jurisdiction is appropriate.

108
Q

A patient who had been injured while being treated at a hospital was planning litigation strategy with her lawyer. The patient and hospital were both citizens of State A. During her hospital stay, the patient suspected that negligence by not only the hospital, but also by her surgeon and a nurse, had contributed to her injuries. The surgeon was a citizen of State A and the nurse was a citizen of State B. After an initial assessment, the lawyer believed that the patient could seek as much as $7,000,000 in damages, for which the hospital, surgeon, and nurse would be jointly and severally liable under controlling state law. Ideally, the patient wanted to sue the hospital, the surgeon, and the nurse in a single lawsuit in federal district court.

How would a federal court address the patient’s potential negligence claims?

A

The plaintiff could join claims against all three potential defendants but the district court would not have subject-matter jurisdiction over all three claims.

plaintiff may join multiple defendants in a single action if: (1) any right to relief is asserted against the defendants jointly, severally, or in the alternative with respect to or arising out of the same transaction, occurrence, or series of transactions or occurrences, and (2) any question of law or fact common to all defendants will arise in the action. Fed. R. Civ. P. 20(a)(2).

A federal district court may exercise subject-matter jurisdiction if a claim: (1) presents a federal question, (2) meets the requirements for diversity jurisdiction (because the parties are completely diverse and the amount in controversy exceeds $75,000), or (3) is within the court’s supplemental jurisdiction. See 28 U.S.C. §§ 1331, 1332, 1367(a). A district court may exercise supplemental jurisdiction if a claim that lacks an independent jurisdictional basis shares a common nucleus of operative fact with another claim that has an independent jurisdictional basis, such that the two claims may be considered part of the same case or controversy. See 28 U.S.C. § 1367(a). However, where the district court’s jurisdiction is based solely on diversity, the court may not exercise supplemental jurisdiction over claims brought by plaintiffs, where doing so is inconsistent with the requirements for diversity. 28 U.S.C. § 1367(b).

Rule 20 allows the patient to join all three potential defendants, because she is arguing that the hospital, surgeon, and nurse are jointly and severally liable for her injuries. The claims against the three potential defendants all arise out of the same hospital visit. All of the claims are based in negligence, so the claims against each defendant will have at least one common issue of fact or law. Therefore, joinder of the three potential defendants is appropriate. Answer options B and D are necessarily incorrect.

Because the potential claims arise under state law, the district court does not have federal-question jurisdiction. The patient and the nurse are citizens of different states, and the amount in controversy of $7,000,000 exceeds the $75,000 threshold, so the district court would have diversity jurisdiction over that claim. However, both the hospital and the surgeon are citizens of State A, like the patient. Therefore, the district court would not have diversity jurisdiction over the patient’s claims against the hospital and the surgeon. Nor could the district court exercise supplemental jurisdiction. Each claim is brought by a plaintiff, in a case where the district court’s sole basis for jurisdiction is diversity. Exercising supplemental jurisdiction would not be consistent with the requirements for diversity (because the patient and each of the defendants are citizens of the same state). Therefore, the district court would not have subject-matter jurisdiction over all three claims.